- PowerScore Staff
- Posts: 5972
- Joined: Mar 25, 2011
- Mon Jan 20, 2014 12:00 am
#49521
Complete Question Explanation
(The complete setup for this game can be found here: lsat/viewtopic.php?t=17097)
The correct answer choice is (A)
The key to this question, and questions #4 and #5, is to Hurdle the UncertaintyTM. When X is added to class 1, the following diagram results:
The remaining variables are V, Z, W, Y, and T. Because the VZ pair and the WY pair cannot be added to the same class, they must occupy spaces in difference classes. But, space is limited in class 1 and class 3. Because no pair can be added to class 2, and in the reverse, no pair can be added to class 1 and class 3 (otherwise the remaining pair would have to both be added to class 2), we can deduce that one pair must be added to classes 1 and 2, and the other pair must be added to classes 2 and 3, as indicated by the straight lines in the diagram below:
This arrangement leaves only one remaining space, and that space must be filled by T:
Accordingly, answer choice (A) is correct.
Questions #3, #4, and #5 each trade on the same principle, but question #3 is considerably harder than questions #4 and #5. After you review the explanations to #4 and #5, return to this question and note how assigning the variable (X, in this case) to class 1 instead of class 3 changed the appearance of the problem. The underlying Hurdle the Uncertainty remains, but this problem is harder to solve because of that difference in variable assignment.
(The complete setup for this game can be found here: lsat/viewtopic.php?t=17097)
The correct answer choice is (A)
The key to this question, and questions #4 and #5, is to Hurdle the UncertaintyTM. When X is added to class 1, the following diagram results:
The remaining variables are V, Z, W, Y, and T. Because the VZ pair and the WY pair cannot be added to the same class, they must occupy spaces in difference classes. But, space is limited in class 1 and class 3. Because no pair can be added to class 2, and in the reverse, no pair can be added to class 1 and class 3 (otherwise the remaining pair would have to both be added to class 2), we can deduce that one pair must be added to classes 1 and 2, and the other pair must be added to classes 2 and 3, as indicated by the straight lines in the diagram below:
This arrangement leaves only one remaining space, and that space must be filled by T:
Accordingly, answer choice (A) is correct.
Questions #3, #4, and #5 each trade on the same principle, but question #3 is considerably harder than questions #4 and #5. After you review the explanations to #4 and #5, return to this question and note how assigning the variable (X, in this case) to class 1 instead of class 3 changed the appearance of the problem. The underlying Hurdle the Uncertainty remains, but this problem is harder to solve because of that difference in variable assignment.
You do not have the required permissions to view the files attached to this post.
Dave Killoran
PowerScore Test Preparation
Follow me on X/Twitter at http://twitter.com/DaveKilloran
My LSAT Articles: http://blog.powerscore.com/lsat/author/dave-killoran
PowerScore Podcast: http://www.powerscore.com/lsat/podcast/
PowerScore Test Preparation
Follow me on X/Twitter at http://twitter.com/DaveKilloran
My LSAT Articles: http://blog.powerscore.com/lsat/author/dave-killoran
PowerScore Podcast: http://www.powerscore.com/lsat/podcast/